Quantcast
  • Register
PhysicsOverflow is a next-generation academic platform for physicists and astronomers, including a community peer review system and a postgraduate-level discussion forum analogous to MathOverflow.

Welcome to PhysicsOverflow! PhysicsOverflow is an open platform for community peer review and graduate-level Physics discussion.

Please help promote PhysicsOverflow ads elsewhere if you like it.

News

PO is now at the Physics Department of Bielefeld University!

New printer friendly PO pages!

Migration to Bielefeld University was successful!

Please vote for this year's PhysicsOverflow ads!

Please do help out in categorising submissions. Submit a paper to PhysicsOverflow!

... see more

Tools for paper authors

Submit paper
Claim Paper Authorship

Tools for SE users

Search User
Reclaim SE Account
Request Account Merger
Nativise imported posts
Claim post (deleted users)
Import SE post

Users whose questions have been imported from Physics Stack Exchange, Theoretical Physics Stack Exchange, or any other Stack Exchange site are kindly requested to reclaim their account and not to register as a new user.

Public \(\beta\) tools

Report a bug with a feature
Request a new functionality
404 page design
Send feedback

Attributions

(propose a free ad)

Site Statistics

205 submissions , 163 unreviewed
5,047 questions , 2,200 unanswered
5,345 answers , 22,709 comments
1,470 users with positive rep
816 active unimported users
More ...

  Liouville's theorem and gravitationally deflected lightpaths

+ 7 like - 0 dislike
1644 views

It is customary in gravitational lensing problems, to project both the background source and the deflecting mass (e.g. a background quasar, and a foreground galaxy acting as a lens) in a plane.

Then, the lensing problem can be regarded as a mapping between the unlensed source plane, and the lensed image plane. In such transformations, the Jacobian evaluated at a point of the source plane, expresses how an infinitesimal area located around that point increases.

Lens mass and mass distribution, relative positions and distances involved give rise to different scenarios. The special case in which the distortions are too small to be resolved by telescopes, is called microlensing regime. Typically, a dark, unseen object like a floating planet, happens to cross transversally in front of a background star.

The image of the background star suffers amplification and distortions that are unresolved, but a change in brightness is detected, with a very typical light curve shape. The measured light curve of a microlensing event can be related to physical parameters of the problem, because the change in brightness of a lensed image can be modelled simply by dividing the area of the lensed image by that of the unlensed source image. If that can be done, it is because the mean surface flux of the image equals that of the source.

That is, gravitational lensing can make a tiny source appear bigger in the sky but in plain terms, every square inch of the image has the same brightness of every square inch of the source. Here comes my question, because that seems to me rather counter-intuitive and, when I try to find a rigorous justification to it, I find the same arcane sentence in each book, in each review, in each paper I have seen:

'Because of Liouville's theorem, gravitational lensing conserves surface brightness'

(... and therefore the magnification is found by dividing the subtended area of the image by that of the source). Every single author I have read, drops that sentence as if it were something very obvious, and quickly goes into other questions.

I have tried to trace-back the origin of the idea, by consulting the bibliography of every book or document in which that thing is stated. Interestingly, I have recognized sort of a fingerprint of obscure points like this one, a patter that is repeated in many of the documents, as if some authors didn't understand and merely copied from each other, developing and personalizing only the parts they understand in between.

I have rigorously developed and resolved every one of the dark points in that pattern, but this one remains unresolved. Is it perhaps something obvious? How is Liouville's theorem applied to photons along null geodesics? I will accept an appropiate link or paper reference as a good answer.

This post imported from StackExchange Physics at 2014-06-14 12:54 (UCT), posted by SE-user Eduardo Guerras Valera
asked Mar 7, 2013 in Theoretical Physics by Eduardo Guerras (435 points) [ no revision ]
See also (related but definitely not a duplicate) physics.stackexchange.com/questions/31534/… - Qmechanic's answer to that question might be helpful to you.

This post imported from StackExchange Physics at 2014-06-14 12:54 (UCT), posted by SE-user Nathaniel
@Nathaniel, thanks (+1). It seem at first sight related. I'll have a closer look, maybe it puts me on the right track.

This post imported from StackExchange Physics at 2014-06-14 12:54 (UCT), posted by SE-user Eduardo Guerras Valera

2 Answers

+ 1 like - 0 dislike

What they're referring to is a property of light from geometrical optics. The conserved property is "entendue" (see wikipedia article), and the constancy of brightness can be demonstrated in a bunch of ways (Hamiltonian optics i.e. Liouville's theorem, second law of thermodynamics as above, etc.).

This post imported from StackExchange Physics at 2014-06-14 12:54 (UCT), posted by SE-user Nanite
answered Sep 3, 2013 by Nanite (50 points) [ no revision ]
You are pointing in the very right direction (+1), the conservation of éntendue is the non-relativistic analogous to this, but what I am looking for is the derivation in the context of gravitational lensing with general relativity, geodesics and so on, where you cannot for instance happily say "energy is conserved across here and there...". I know this issue is waiting for me, I will have to stop sooner or later and spend the necessary time to try to understand and derive it by myself. But I would like to see the original derivation all authors are ignoring, because now I am short on time.

This post imported from StackExchange Physics at 2014-06-14 12:54 (UCT), posted by SE-user Eduardo Guerras Valera
Yeah, I figured you were looking for the real derivation. I wish I could help but my classical-mechanics-fu is not so strong.

This post imported from StackExchange Physics at 2014-06-14 12:54 (UCT), posted by SE-user Nanite
(I would guess that the original non-relativistic proof (if you can find it), if put into strictly hamiltonian terms, should work well enough because the hamiltonian mechanics can be used also to model relativistic motion, with suitable choice of coordinates.)

This post imported from StackExchange Physics at 2014-06-14 12:54 (UCT), posted by SE-user Nanite
By the way, ouch, the correct spell is "étendue". It is a frech word meaning sort of "extended" or "extensive". I also spelled it wrong in my comment above, which cannot be edited any more (unlike your answer, that can be edited easily)

This post imported from StackExchange Physics at 2014-06-14 12:54 (UCT), posted by SE-user Eduardo Guerras Valera
+ 0 like - 0 dislike

This is just the second law of thermodynamics.

Suppose you have some big source black body and some little target black body. You build a whole bunch of geometric optics to focus light from the source to the target (e.g. mirrors behind the target and lenses in front of it). Eventually an observer standing on the target looks up and sees light from the source at every point in the sky.

All this incoming light heats the target blackbody up until it's radiating light away as fast as it's coming in. That means the target comes up to the same temperature as the source. If you could increase the flux per unit solid angle, the target would get hotter than the source, breaking the second law.

Is this enough, or did you want a technical analysis of the lensing equation?

This post imported from StackExchange Physics at 2014-06-14 12:54 (UCT), posted by SE-user Mark Eichenlaub
answered Mar 7, 2013 by Mark Eichenlaub (100 points) [ no revision ]
I am afraid it is not enough. (i) I don't grasp why the observer standing at the target cannot simply receive less light from, say, the mirrors standing behind, but more light from the front lens, thus having parts of the image brighter at the cost of having other parts of the image darker. (ii) In real problems, the observer receives only part of the flux, the rest goes to another directions. I don't see why a spatially extended image cannot be dimmer (that would be my guess). So the technical analysis of the lensing equation would be fantastic, if you feel like writing it...

This post imported from StackExchange Physics at 2014-06-14 12:54 (UCT), posted by SE-user Eduardo Guerras Valera
I had read too about the violation of the 2nd principle if you could concentrate light in too small a point with a magnifying glass, but I don't see how that translates to this question. That is why I want to find the mathematical details, that will surely give rise to the correct understanding.

This post imported from StackExchange Physics at 2014-06-14 12:54 (UCT), posted by SE-user Eduardo Guerras Valera
The second-law argument does work. For example if there is less light from the mirror behind the black body, just paint the back of it perfect white. Re: Liouville's theorem in geometric optics; I might have some time for that later on but I can't just do it in a couple minutes off the top of my head like this above argument.

This post imported from StackExchange Physics at 2014-06-14 12:54 (UCT), posted by SE-user Mark Eichenlaub

Your answer

Please use answers only to (at least partly) answer questions. To comment, discuss, or ask for clarification, leave a comment instead.
To mask links under text, please type your text, highlight it, and click the "link" button. You can then enter your link URL.
Please consult the FAQ for as to how to format your post.
This is the answer box; if you want to write a comment instead, please use the 'add comment' button.
Live preview (may slow down editor)   Preview
Your name to display (optional):
Privacy: Your email address will only be used for sending these notifications.
Anti-spam verification:
If you are a human please identify the position of the character covered by the symbol $\varnothing$ in the following word:
p$\hbar$ysicsO$\varnothing$erflow
Then drag the red bullet below over the corresponding character of our banner. When you drop it there, the bullet changes to green (on slow internet connections after a few seconds).
Please complete the anti-spam verification




user contributions licensed under cc by-sa 3.0 with attribution required

Your rights
...